Найдите предел функции lim: Правило Лопиталя онлайн

2}=\infty$%. Пусть $%x=t,y=-t:lim=0.$% Значит, предела нет.

ссылка

отвечен 25 Июн ’12 18:52

dmg3
750●1●10●48

изменен 25 Июн ’12 18:58

Пусть, $%M$%-некоторое положительное число, $%|x| \gt M,\; |y| \gt M$%.
Это означает, что
либо $%x \gt M,\; y \gt M$%, либо $%x \gt M,\; y \lt -M$%, либо $%x \lt -M,\; y \gt M$%, либо $%x \lt -M,\; y \lt -M$%.

2}$$ существует и равен нулю.

ссылка

отвечен 2 Июл ’12 16:37

Андрей Юрьевич
4.0k●9●19

Ответ – нуль.

0<(x+y)/(x2-xy+y2)=(x+y)/((x-y)2+xy)<(x+y)/(xy)=1/y+1/x-> 0.

По лемме о двух мажорантах к нулю стремится и функция, стоящая внутри неравенств.

ссылка

отвечен 2 Июл ’12 12:05

org/Person”>Vvk
11●2

изменен 2 Июл ’12 13:45

Можно еще представить в полярной системе координат: $%x=r\cos \varphi,y=r\sin \varphi$%, предел берется при $%r\to+\infty$%. Тогда “подпредельное” выражение принимает вид $%\frac{\cos \varphi+\sin \varphi}{r(1-\cos \varphi\sin \varphi)}$% = $%\frac{\cos \varphi+\sin \varphi}{r(1-\sin 2\varphi/2)}$%. Числитель не превосходит 2, а знаменатель не меньше, чем $%r/2$%, значит, дробь не больше $%4/r$%, откуда и следует, что она стремится к 0.

ссылка

отвечен 4 Июл ’12 0:45

org/Person”>DocentI
9.9k●2●21●52

Ваш ответ

Если вы не нашли ответ, задайте вопрос.

2y}{x-y} $

спросил

Изменено 1 год, 6 месяцев назад

Просмотрено 269 ​​раз

$\begingroup$

Я пытаюсь определить предел следующей функции по мере приближения к (0,0): $$\lim_{(x,y)\rightarrow(0,0)} \frac{x^2y}{x-y} $$ 92\тета соз\тета}{соз\тета-грех\тета}$$ Я не могу доказать, что предел приближается к 0, поскольку часть, зависящая от $\theta$ в знаменателе, может варьироваться и быть сколь угодно малой, в то время как $\theta$ приближается к $\frac{\pi}{4} $, что означает, что я не могу его связать.

Я безуспешно пытался найти функцию верхней границы, приближающуюся к 0, чтобы использовать теорему “Сэндвич”.

Буду признателен за помощь.

  • исчисление
  • пределы
  • многомерное исчисление 9{2}\theta_{n})(2/n)}{\sin(2/n)}\dfrac{n}{2}. \конец{выравнивание*} Пусть $r_{n}=1/\sqrt[4]{n}$ и используем тот факт, что $\dfrac{2/n}{\sin(2/n)}\rightarrow 1$, легко увидеть что $f(x_{n},y_{n})\rightarrow\infty$.

    $\endgroup$

    Зарегистрируйтесь или войдите в систему

    Зарегистрируйтесь с помощью Google

    Зарегистрироваться через Facebook

    Зарегистрируйтесь, используя электронную почту и пароль

    Опубликовать как гость

    Электронная почта

    Требуется, но никогда не отображается

    Опубликовать как гость

    Электронная почта

    Требуется, но не отображается

    Нажимая «Опубликовать свой ответ», вы соглашаетесь с нашими условиями обслуживания, политикой конфиденциальности и политикой использования файлов cookie

    Видео-вопрос: нахождение значения предела на графике

    Стенограмма видео

    На следующем рисунке представлен график функции 𝑓. Что график говорит о значении предела, когда 𝑥 приближается к трем из 𝑓 из 𝑥?

    Нам дан график функции 𝑓. И нам нужно использовать это, чтобы узнать информацию о пределе, когда 𝑥 приближается к трем из 𝑓 из 𝑥. И первое, что нам нужно вспомнить, это то, что мы подразумеваем под пределом, когда 𝑥 приближается к трем из 𝑓 из 𝑥. Нам нужно напомнить, что мы говорим, что предел приближения 𝑥 к 𝑎 из 𝑓 из 𝑥 равен некоторому конечному значению 𝐿, если значение 𝑓 из 𝑥 приближается к 𝐿, когда наши значения 𝑥 приближаются к 𝑎 с обеих сторон. И мы хотим знать, что график говорит нам о пределе, когда 𝑥 приближается к трем из 𝑓 из 𝑥.

    Итак, чтобы сделать это, нам нужно будет задать вопрос, что происходит, когда 𝑥 приближается к трем с каждой стороны к нашим выходам 𝑓 из 𝑥? И для этого нам дан график нашей функции. Помните, 𝑦-координата функции сообщает нам выход 𝑓 из 𝑥, а 𝑥 является входом нашей функции, и мы хотим знать, что случилось с выходами нашей функции, когда наши входы приближаются к трем с каждой стороны. Итак, давайте начнем с того, что отметим 𝑥 равным трем на нашей диаграмме, и мы можем увидеть кое-что очень интересное в нашей функции, когда 𝑥 равно трем. Из кривой видно, что она не определена, когда 𝑥 равно трем. Это представлено полым кругом, поэтому у нас может возникнуть соблазн сказать, что этот предел не существует, потому что наша функция не определена в этой точке.

    Однако давайте еще раз взглянем на наше определение. Мы хотим знать, что происходит с нашими выходами 𝑓 из 𝑥, когда 𝑥 приближается к 𝑎. И когда мы говорим, что 𝑥 приближается к 𝑎, мы имеем в виду, что 𝑥 все ближе и ближе приближается к 𝑎; 𝑥 никогда не равно 𝑎. Таким образом, наша функция, не определенная в этот момент, не обязательно означает, что предел не определен. Итак, давайте посмотрим, что происходит с нашей функцией, когда 𝑥 приближается к трем. Начнем с того, что 𝑥 приближается к трем справа. Есть несколько разных способов сделать это. Мы могли бы подставить разные значения 𝑥 в наше уравнение. Начнем с того, что 𝑥 равно пяти. Когда 𝑥 равно пяти, мы можем видеть, что 𝑦-координата нашей кривой составляет около 0,5, поэтому 𝑓 пяти примерно равно 0,5.

    Помните, мы хотим посмотреть, что произойдет с 𝑓 из 𝑥, когда 𝑥 приблизится к трем. Поэтому нам нужно выбирать все более и более близкие значения к трем, поэтому мы сделаем то же самое, но на этот раз с 𝑥 равным четырем. И когда 𝑥 равно четырем, мы видим, что наша кривая имеет 𝑦-координату, которая приблизительно равна минус 3,5. Таким образом, 𝑓 из четырех примерно равно минус 3,5. Мы хотим продолжать выбирать все более и более близкие значения к трем. И мы можем видеть на нашей кривой, что по мере того, как мы это делаем, наши ценности становятся все ближе и ближе к этому пустому кругу. Так как 𝑥 приближается к трем справа, наши 𝑦-координаты все ближе и ближе к отрицательным четырем. Поэтому мы хотели бы выбрать наше значение 𝐿 равным минус четырем. Однако нам нужно проверить, что то же самое происходит, когда 𝑥 приближается к трем слева.

    Итак, давайте выберем некоторые значения 𝑥 меньше трех.

Оставить комментарий